How to Correctly Differentiate the Acceleration Equation in N-Body Simulations?

Click For Summary
The discussion focuses on the differentiation of the acceleration equation in the context of the Hermite integration scheme for N-body simulations. The user struggles with correctly evaluating the derivative of acceleration, specifically in applying the quotient rule and the chain rule. They initially confuse scalar and vector quantities but ultimately find that expressing the change in radius in terms of the velocity vector and the angle between vectors clarifies the differentiation process. The correct approach involves maintaining scalar quantities throughout the differentiation and using the dot product to relate the vectors. This method leads to the accurate formulation of jerk in the context of N-body simulations.
cc94
Messages
19
Reaction score
2

Homework Statement


I was reading about the Hermite integration scheme for N-body simulations, as seen here: http://www.artcompsci.org/kali/vol/two_body_problem_2/ch11.html#rdocsect76

This scheme uses jerk, the time rate of change of acceleration. The problem is that I don't know how to correctly evaluate the derivative of the acceleration equation shown there.

Homework Equations



\vec{a} = \frac{GM}{r^2}\vec{\hat{r}}
\vec{j} = GM\left(\frac{\vec{v}}{r^3} - 3 \frac{(\vec{r} \cdot \vec{v})\vec{r}}{r^5}\right)

The Attempt at a Solution


First I rewrote the unit vector in a to get:
\vec{a}= \frac{GM\vec{r} }{r^3}

To differentiate, I applied the quotient rule:
\frac{d\vec{a}}{dt} = GM\frac{r^3\frac{d\vec{r}}{dt} - \vec{r}\frac{dr^3}{dt}}{r^6}

The first term is obviously v/r^3, but the second term is where I'm confused. I can get the right answer by using the chain rule:

\frac{\frac{dr^3}{dt}}{r^6} = \frac{3r^2\frac{d\vec{r}}{dt}}{r^6} = \frac{3\vec{r}\frac{d\vec{r}}{dt}}{r^5} = \frac{3(\vec{r}\cdot\vec{v})}{r^5}

but I'm just arbitrarily changing the scalar r into the vector r. What's the right way to do this step?
 
Last edited:
Physics news on Phys.org
The answer is correct, but the steps are all wrong: the first because it equates a scalar to a vector, the second because it contains a meaningless multiplication of two vectors, and the third because it then somehow gets back to a scalar quantity.
Instead, keep the amounts as scalars all the way along. Start with:
$$\frac{d(r^3)}{dt}=3r^2\frac{dr}{dt}$$
Then draw a diagram and try to express ##\frac{dr}{dt}## in terms of ##\vec v## and ##\vec r##. You may find it convenient to use ##\hat r##, the unit vector in the direction of ##\vec r##, along the way.
 
Ok so, if we draw \vec{r} = r\hat{r} on the plane, for instance, then the change in length will depend on which direction v is pointing. In the limit of a small time step dt, the change of length r is equal to v*dt*cos(θ), where θ is the angle between v and r. I.e.
\frac{dr}{dt} = vcos\theta

So we pull one of the 'r's from in front to get r*v*cos(θ), which is indeed equal to the dot product r·v. The remaining r cancels one in the bottom to get the r^5 term.
 
Yes, that's it!
 
Question: A clock's minute hand has length 4 and its hour hand has length 3. What is the distance between the tips at the moment when it is increasing most rapidly?(Putnam Exam Question) Answer: Making assumption that both the hands moves at constant angular velocities, the answer is ## \sqrt{7} .## But don't you think this assumption is somewhat doubtful and wrong?

Similar threads

  • · Replies 9 ·
Replies
9
Views
2K
  • · Replies 7 ·
Replies
7
Views
2K
  • · Replies 7 ·
Replies
7
Views
2K
Replies
3
Views
3K
  • · Replies 13 ·
Replies
13
Views
3K
  • · Replies 7 ·
Replies
7
Views
2K
  • · Replies 2 ·
Replies
2
Views
2K
  • · Replies 5 ·
Replies
5
Views
2K
  • · Replies 1 ·
Replies
1
Views
1K
  • · Replies 2 ·
Replies
2
Views
2K